Hiển thị các bài đăng có nhãn Bất đẳng thức. Hiển thị tất cả bài đăng
Hiển thị các bài đăng có nhãn Bất đẳng thức. Hiển thị tất cả bài đăng

Thứ Sáu, 26 tháng 6, 2015

Chủ Nhật, 8 tháng 6, 2014

Tìm GTNN của $F=\frac{1}{1+a^6}+\frac{2}{1+b^3}+\frac{3}{1+c^2}+6\sqrt{1+abc(abc-1)}$

Cho $a,b,c\ge 1$ tìm GTNN của $$F=\frac{1}{1+a^6}+\frac{2}{1+b^3}+\frac{3}{1+c^2}+6\sqrt{1+abc(abc-1)}$$
Trích đề thi thử TT BDVH 218 Lý Tự Trọng

Thứ Năm, 5 tháng 6, 2014

Thứ Ba, 27 tháng 5, 2014

Thứ Sáu, 23 tháng 5, 2014

Tìm giá trị nhỏ nhất biểu thức: $$P=\sqrt{1+\frac{24(y+z-x)}{x}}+\sqrt{1+\frac{24(z+x-y)}{y}}+\sqrt{1+\frac{24(x+y-z)}{z}}$$

Cho $x,y,z$ là độ dài 3 cạnh tam giác. Tìm giá trị nhỏ nhất biểu thức: $$P=\sqrt{1+\frac{24(y+z-x)}{x}}+\sqrt{1+\frac{24(z+x-y)}{y}}+\sqrt{1+\frac{24(x+y-z)}{z}}$$
Đề thi thử New-study lần 12

Chủ Nhật, 11 tháng 5, 2014

Câu BĐT đề người thầy lần 6

Cho $a,b,c>0$ thỏa $(a+b-c)\left(\frac{1}{a}+\frac{1}{b}-\frac{1}{c} \right )=4$. Tìm GTNN của $$P=(a^3+b^3+c^3)\left(\frac{1}{a^3}+\frac{1}{b^3}+\frac{1}{c^3} \right )$$
Đề người thầy lần 6 2014

Thứ Bảy, 12 tháng 4, 2014

Tìm giá trị nhỏ nhất của biểu thức: $$P=\frac{(x+y-z)^2}{x^2+y^2+z^2+2xy}+\frac{(y+z-x)^2}{x^2+y^2+z^2+2yz}+\frac{(z+x-y)^2}{x^2+y^2+z^2+2xz}$$

Cho các số thực dương $x,y,z$ thỏa $x+y+z=3$. Tìm giá trị nhỏ nhất của biểu thức: $$P=\frac{(x+y-z)^2}{x^2+y^2+z^2+2xy}+\frac{(y+z-x)^2}{x^2+y^2+z^2+2yz}+\frac{(z+x-y)^2}{x^2+y^2+z^2+2xz}$$

Đề thi thử lần 2 chuyên Lê Quý Đôn - Đà Năng
Lời giải

Chủ Nhật, 9 tháng 6, 2013

Tìm GTNN của biểu thức $P=x^2+y^2+2z+\frac{40}{\sqrt{y+z+1}}+\frac{40}{\sqrt{x+3}}$

Cho $x,y,z$ thực thỏa mãn $x^2+y^2+z^2+2xy\le 2(x+y+z)$. Tìm GTNN của biểu thức $$P=x^2+y^2+2z+\frac{40}{\sqrt{y+z+1}}+\frac{40}{\sqrt{x+3}}$$

Trích đề thi thử THPT Chuyên Lê Hồng Phong THPT

Chủ Nhật, 16 tháng 12, 2012

Chứng minh $$a^3b+b^3c+c^3a \ge a^2b^2+b^2c^2+c^2a^2$$

Cho tam giác $ABC, BC=a,CA=b,AB=c$. Chứng minh $$a^3b+b^3c+c^3a \ge a^2b^2+b^2c^2+c^2a^2$$
Ta có nhận xét sau: Với mọi $M,N$ trong tam giác $ABC$ thì $$MA.\sin\angle BNC+MB.\sin \angle CNA+MC.\sin \angle ANB\ge NA.\sin \angle BNC+NB.\sin \angle CNA+NC.\sin \angle ANB$$
Tiếp đó cho $M$ và N là hai điểm Broca của tam giác, chú y$\angle BNC=180^0-B;\angle CNA=180^0-C;\angle ANB=180^0-A$.
Từ đây dễ dàng có điều cần chứng minh.

Chứng minh $ab+bc+ac\ge 1$

Cho $x,y,z$ là các số thực dương thỏa mãn $x+y+z=1$. Đặt $$a=\sqrt{y^2+yz+z^2};b=\sqrt{z^2+xz+x^2};c=\sqrt{x^2+xy+y^2}$$ Chứng minh $ab+bc+ac\ge 1$
Giả sử cho điểm $M$ bất kì trong mặt phẳng.
Dựng các điểm $A,B,C$ sao cho $MA=x;MB=y;MC=z$ và $\angle AMB=\angle BMC=\angle CMA=120^0$
Khi đó $MA$ là điểm Toricelli trong tam giác ABC có 3 cạnh $BC=a;CA=b;AB=c$. Ta cần chứng minh $$ab+bc+ac\ge 1=MA+MB+MC =\sqrt{\frac{a^2+b^2+c^2}{2}+2S\sqrt{3}}$$
Hay $$a^2+b^2+c^2 \ge 4S\sqrt{3}+(a-b)^2+(b-c)^2+(c-a)^2$$
Đây chính là bất đẳng thức Finsler-Hadwinger
Vậy ta có điều cần chứng minh. $\blacksquare$

Thứ Ba, 11 tháng 12, 2012

Tìm GTLN, GTNN của biểu thức: $P=\frac{a^2+b^2+c^2}{ab+bc+ac}$

chotam giác ABC  ngoại tiếp đường tròn tâm I, có trọng tâm G. G nằm trong (I). Gọi a, b, c lần lượt là độ dài các cạnh BC, CA, AB của ∆ABC. Tìm GTLN, GTNN của biểu thức:
$P=\frac{a^2+b^2+c^2}{ab+bc+ac}$
Lời giải
GTNN
Ta có $a^2+b^2+c^2 \ge ab+bc+ac$ do đó $P\ge 1$
Dấu "=" xảy ra khi $a=b=c$ khi đó tam giác $ABC$ đều.
GTLN
Gọi $p$ và $r$ theo thứ tự là nửa chu vi và bán kính đường tròn nội tiếp tam giác $ABC$. Theo định lý Pythagore ta có
$$IA^2+IB^2+IC^2=(p-a)^2+(p-b)^2+(p-c)^2+3r^2 \,\,\,\,(1)$$
Lại có $$IA^2+IB^2+IC^2=(\overrightarrow{IG}+\overrightarrow{GA})^2+(\overrightarrow{IG}+\overrightarrow{GB})^2+(\overrightarrow{IG}+\overrightarrow{GC})^2=3IG^2+GA^2+BG^2+CG^2\,\,\, (2)$$
Do $\overrightarrow{GA}+\overrightarrow{GC}+\overrightarrow{GB}=\overrightarrow{0}$
Từ (1) (2) và để ý rằng $GA^2+BG^2+CG^2=\frac{a^2+b^2+c^2}{3}$ theo công thức đường trung tuyến ta có $$(p-a)^2+(p-b)^2+(p-c)^2+3r^2 =3IG^2+\frac{a^2+b^2+c^2}{3}\,\,\,\, (3)$$
Vì $I$ nằm trong hình tròn $(I) $ nên $IG \le r$
Từ (3) ta có $$(p-a)^2+(p-b)^2+(p-c)^2+3r^2 =3IG^2+\frac{a^2+b^2+c^2}{3}$$
Do đó $P\le \frac{6}{5}$
Đẳng thức xảy ra tại $IG =r$. khi đó $G$ nằm trên đường tròn (I) chẳng hạn đối với tam giác có độ dài ba cạnh là $5;10;13\blacksquare$

Thứ Hai, 10 tháng 12, 2012

India RMO 2012

Cho $a,b,c$ thực dương thỏa mãn $abc(a+b+c)=3$. Chứng minh $(a+b)(b+c)(c+a)\ge 8$
India RMO 2012
Lời giải
Ta có $(ab+bc+ac)^2 \ge 3abc(a+b+c)\Rightarrow ab+bc+ac\ge 3$
$$(a+b)(b+c)(c+a) \ge \frac{8}{9}(a+b+c)(ab+bc+ac)\ge \frac{8.9}{9}=8$$
Do $a+b+c\ge \sqrt{3(ba+bc+ac)}$
Dấu "=" xảy ra khi $a=b=c=1$

Cho $a,b$ là hai số thực dương thỏa mãn $a+b=1$. Chứng minh $a^ab^b+a^bb^a\le 1$
India RMO 2012
Lời giải
Áp dụng bất đẳng thức AM-GM suy rộng ta có $$a^ab^b+a^bb^a \le a^2+b^2+2ab=(a+b)^2 =1$$
Dấu "=" xảy ra khi $a=b=\frac{1}{2}$

Thứ Sáu, 7 tháng 12, 2012

Chứng minh rằng $a^3+b^3+c^3+3abc \ge ab(a+b)+bc(b+c)+ac(a+c). $

Cho $a,b,c \ge 0$. Chứng minh rằng $a^3+b^3+c^3+3abc \ge ab(a+b)+bc(b+c)+ac(a+c). $
Lời giải
Đặt $f(a;b;c)=a^3+b^3+c^3+3abc-[ab(a+b)+bc(b+c)+ca(c+a)]$
Điểm cực trị $f(a;b;c)$ là nghiệm của hệ
$$\frac{\partial f}{\partial a}=\frac{\partial f}{\partial b}=\frac{\partial f}{\partial c}$$
$$\Leftrightarrow \left\{\begin{matrix} 3(a^2+bc)=2ab+2ac+b^2+c^2\\ 3(b^2+ac)=2ab+2bc+a^2+c^2\\ 3(c^2+ab)=2ac+2bc+a^2+b^2 \end{matrix}\right.$$
Cộng theo vế ta có $a^2+b^2+c^2=ab+bc+ca\Leftrightarrow a=b=c$ khi đó $f(a;b;c)=0$
Xét TH $a=0:b^3+c^3\geq bc(b+c)\Leftrightarrow (b+c)(b-c)^3\geq 0$
Dấu đẳng thức chỉ xảy ra $\Leftrightarrow a=b=c$ hoặc $a=0,b=c$ và hoán vị tương ứng

Thứ Tư, 5 tháng 12, 2012

Chứng minh $$\frac{a}{a^2+3}+\frac{b}{b^2+3}+\frac{c}{c^2+3}\leq \frac{3}{4}$$

Cho các số dương $a,b,c$ thỏa mãn $abc=1$. Chứng minh
$$\frac{a}{a^2+3}+\frac{b}{b^2+3}+\frac{c}{c^2+3}\leq \frac{3}{4}$$
Lời giải 
Do $abc=1$ nên lấy Logarith Napier ta có:
$$\ln a+\ln b+\ln c=0$$
Ta sẽ chứng minh bất đẳng thức phụ sau:
$$\frac{a}{a^2+3}\leq \frac{1}{4}+\frac{1}{8}.\ln a$$
Thật vậy xét $f(a)=\frac{a}{a^2+3}- \frac{1}{8}.\ln a$
Ta có : $f'(a)=\frac{3-a^2}{(a^2+3)^2}- \frac{1}{8a}\\ =\frac{(1-a)(a+1)(a^2+15)}{8a(3+a^2)^2}$
Vậy $f'(a)$ đổi dấu từ dương sang âm khi $a=1$. $f(a)_{Max}=f(1)=0$
Thiết lập các bất đẳng thức tương tự và cộng lại ta có:
$$\frac{a}{a^2+3}+\frac{b}{b^2+3}+\frac{c}{c^2+3}\leq \frac{3}{4}+\frac{1}{8}(\ln a+\ln b+\ln c)=\frac{3}{4}$$
Ta có điều phải chứng minh. Đẳng thức xảy ra tại $a=b=c=1$

Chủ Nhật, 2 tháng 12, 2012

Tìm giá trị lớn nhất của biểu thức : $P = \frac{{25\left( {xy + yz + zx} \right)}}{{2{x^3} + 3{y^2} + 6z + 1}} - \sqrt[4]{{xyz}}$

Cho $3$ số thực dương $x,y,z$ thỏa mãn điều kiện $x+y+z=3$. Tìm giá trị lớn nhất của biểu thức : $$P = \frac{{25\left( {xy + yz + zx} \right)}}{{2{x^3} + 3{y^2} + 6z + 1}} - \sqrt[4]{{xyz}}$$
Lời giải
Áp dụng bất đẳng thức AM-GM ta có: $x+y+z \geq 3 \sqrt[3]{xyz} \Rightarrow xyz \le 1 \Rightarrow \sqrt[4]{xyz} \geq xyz$
Tiếp tục áp dụng bất đăng thức AM-GM ta có:
$x^3+x^3+1 \geq 3x^2 \Rightarrow 2x^3+3y^2+6z \geq 3x^2+3+3y^2+3+6z-6 \geq 12$
$ \Rightarrow P \le \dfrac{25}{12}(xy+yz+zx)- xyz$
Áp dụng bất đẳng thức Schur  $(x+y-z)(y+z-x)(z+x-y) \le xyz$
hay $(3-2z)(3-2x)(3-2y) \le xyz \Rightarrow xy+yz+zx \le \dfrac{9}{4}+ \dfrac{3}{4}xyz$
$ \Rightarrow \dfrac{25}{12}(xy+yz+zx)- xyz \le \dfrac{75}{16}+ \dfrac{9}{16}xyz \le \dfrac{21}{4}$
Vậy $MaxP= \dfrac{21}{4}$ khi $x=y=z=1$.

Thứ Sáu, 30 tháng 11, 2012

Cho $x,y,z$ thực dương thỏa $xyz=1$. Tìm GTNN của $P=x^3y^3+y^3z^3+z^3x^3-6(x+y+z)$

Cho $x,y,z$ thực dương thỏa $xyz=1$. Tìm GTNN của $$P=x^3y^3+y^3z^3+z^3x^3-6(x+y+z)$$
Nghệ An MO 2009
Lời giải
Ta có: $P=\frac{1}{x^3}+\frac{1}{y^3}+\frac{1}{z^3}-6(x+y+z)$
-Xét hàm: $L=\frac{1}{x^3}+\frac{1}{y^3}+\frac{1}{z^3}-6(x+y+z)+\lambda (xyz-1)$
-Điểm cực trị là nghiệm của hệ:
$\begin{cases}
& \ L'_x=L'_y=L'_z=0  \\
& \ xyz=1
\end{cases}\\\Leftrightarrow \begin{cases}
& \ -3.\frac{1}{x^4}-6+\lambda yz=0  \\
& \ -3.\frac{1}{y^4}-6+\lambda xz=0  \\
& \ -3.\frac{1}{z^4}-6+\lambda xy=0 \\
& \ xyz=1 
\end{cases}\\\Leftrightarrow \begin{cases}
& \ \frac{1}{x^3}+2x=\frac{1}{y^3}+2y=\frac{1}{z^3}+2z=\frac{\lambda }{3}(*)  \\
& \ xyz=1
\end{cases}$
-Xét $f(x)=\frac{1}{x^3}+2x\\\Rightarrow f''(x)=\frac{12}{x^5}>0 \forall x>0$
Do đó, trong 3 số x;y;z, có 1 cặp số bằng nhau!
-Với $x=y$, biến đổi, ta được:
$(x^3-1)(x^6+x^3-2x+1)=0$
Chứng minh được $x=y=z=1$
-Hệ có nghiệm duy nhất: $(x;y;z)=(1;1;1)$
-Ta có:
$P(1;1;1)=-15$
$P(1;0,5;2)=-11,875>-15$
Nên $P_{min}=-15\Leftrightarrow x=y=z=1$

Chứng minh rằng: $\frac{a^2}{b}+\frac{b^2}{c}+\frac{c^2}{a}\geq a+b+c+\frac{4(a-b)^2}{a+b+c}$

Với $a$, $b$, $c>0$ chứng minh rằng:
$$\frac{a^2}{b}+\frac{b^2}{c}+\frac{c^2}{a}\geq a+b+c+\frac{4(a-b)^2}{a+b+c}$$
Lời giải 
Bất đẳng thức cần chứng minh tương đương
$$(\frac{a^2}{b}-2a+b)+(\frac{b^2}{c}-2b+c)+(\frac{c^2}{a}-2c+a)=\frac{(a-b)^2}{b}+\frac{(b-c)^2}{c}+\frac{(c-a)^2}{a}\geq \frac{4(a-b)^2}{a+b+c}$$
Ta có: $$\frac{(a-b)^2}{b}+\frac{(b-c)^2}{c}+\frac{(c-a)^2}{a}\geq \frac{(|a-b|+|b-c|+|c-a|)^2}{a+b+c}$$
Ta cần chứng minh $(|a-b|+|b-c|+|c-a|)^2\geq 4(a-b)^2$
Dễ thấy $$|b-c|+|c-a|\geq |a-b|\Rightarrow (2|a-b|)^2\geq 4(a-b)^2$$

Điều này đúng suy ra điều phải chứng minh $\blacksquare$

Bất đẳng thức hình học

Với các bạn yêu thích Hình học,bài toán dưới đây là 1 vài bài toán quen thuộc:
$$\boxed{a^2+b^2+c^2 \ge 4\sqrt{3}S }$$
Đây được gọi là Bất đẳng thức Weitzenbock.
Chứng minh
Sử dụng phép thế Ravi đặt $a=y+z; b=x+z; c=x+y$ với $x,y,z>0$ . Bất đẳng thức cần chứng minh
$$\Leftrightarrow \begin{pmatrix}
(y+z)^2+(x+z)^2+(x+y)^2
\end{pmatrix}^2\ge 48(x+y+z)xyz$$
Thật vậy ta có $$VT\ge (4xy+4xz+4yz)^2\ge 16(xy+xz+yz)^2\ge 16.3xyz(x+y+z)=48xyz(x+y+z)$$
Vậy bài toán được chứng minh. $\blacksquare$
 Và 1 bất đẳng thức chặt hơn,được gọi là Bất đẳng thức Finsler-Hadwinger
$$\boxed{a^2+b^2+c^2 \ge 4\sqrt{3}S+(a-b)^2+(b-c)^2+(c-a)^2}$$
Chứng minh
Khai triển ra ta có bất đẳng thức trên viết lại thành $2(ab+bc+ac)-(a^2+b^2+c^2)\ge 4\sqrt{3}$
Cách 1: Sử dụng phép thế Ravi ta đặt $a=y+z;b=x+z;c=x+y$ với $x,y,z>0$, bất đẳng thức cần chứng minh $\Leftrightarrow xy+xz+yz\ge \sqrt{3xyz(x+y+z)}$
Ta có $(xy+xz+yz)^2-3xyz(x+y+z)=\frac{(xy-yz)^2+(yz-xz)^2+(xz-xy)^2}{2}\ge 0$
Bất đẳng thức này luôn đúng.
Cách 2:$$\frac{2ab+2bc+2ac-(a^2+b^2+c^2)}{4S}=tg\frac{A}{2}+tg\frac{B}{2}+tg\frac{C}{2}$$ vì hàm số $tg x$ lồi trên $(0;\frac{\pi}{2})$ nên theo BĐT Jensen ta có $$\frac{2ab+2bc+2ac-(a^2+b^2+c^2)}{4S} \geq 3tg \begin{pmatrix}
\frac{\frac{A}{2}+\frac{B}{2}+\frac{C}{2}}{3}
\end{pmatrix}=\sqrt{3}$$
Vậy bài toán được chứng minh. $\blacksquare$

Cho $a_1;b_1;c_1$ là độ dài cạnh tam giác $A_1B_1C_2$ có diện tích $S_1$ và $a_2,b_2,c_2$ là độ dài 3 cạnh tam giác $A_2B_2C_2$ có diện tích $S_2$. Chứng minh: $$\boxed{a_1^2(b_2^2+c_2^2-a_2^2)+b_1^2(c_2^2+a_2^2-b_2^2)+c_1^2(a_2^2+b_2^2-c_2^2)\ge 16S_1S_2}$$
Đây là mở rộng của BĐT Weitzenbock.
Chứng minh:
Cách 1:
Bổ đề: Bất đẳng thức Aczela
Bổ đề: Cho $2n$ số thực không âm $a_1;a_2;...;a_{n};b_1;b_2;...b_{n}$ và 2 số thực không âm thỏa mãn:
$$\alpha^2 \ge \sum_{k=1}^{n}a_{k}^2$$
$$\beta^2 \ge \sum_{k=1}^{n}b_{k}^2$$
Chứng minh:
$$\left(\alpha^2- \sum_{k=1}^{n}a_{k}^2 \right)\left(\beta^2-\sum_{k=1}^{n}b_{k}^2 \right) \le \left(\alpha.\beta -\sum_{k=1}^{n}a_{k}b_{k} \right)^2$$

Chứng minh:
Theo C-S:
$$\alpha.\beta \ge \sqrt{\left(\sum_{k=1}^{n}a_{k}^2 \right)\left(\sum_{k=1}^{n}b_{k}^2 \right)} \ge \sum_{k=1}^{n}a_{k}b_{k}$$
Viết lại BĐT dưới dạng:
$$\sum_{k=1}^{n}a_{k}b_{k} +\sqrt{\left(\alpha^2- \sum_{k=1}^{n}a_{k}^2 \right)\left(\beta^2-\sum_{k=1}^{n}b_{k}^2 \right)} \le \alpha.\beta$$
Theo C-S:
$$VT \le \sqrt{\left(\sum_{k=1}^{n}a_{k}^2 \right)\left(\sum_{k=1}^{n}b_{k}^2 \right)}+\sqrt{\left(\alpha^2- \sum_{k=1}^{n}a_{k}^2 \right)\left(\beta^2-\sum_{k=1}^{n}b_{k}^2 \right)} \le \sqrt{\alpha^2.\beta^2}=\alpha.\beta=VP$$
Vậy bổ đề đã được chứng minh.
Quay trở lại bài toán,bằng công thức Herone:
$$4S_1=\sqrt{2\sum_{cyc}a_{1}^2b_{1}^2 -\sum_{cyc}a_{1}^4}=\sqrt{\left(\sum_{cyc}a_{1}^2 \right)^2-2\sum_{cyc}a_{1}^4}$$
$$4S_2=\sqrt{2\sum_{cyc}a_2^2b_2^2 -\sum_{cyc}a_2^4}=\sqrt{\left(\sum_{cyc}a_{2}^2 \right)^2-2\sum_{cyc}a_{2}^4}$$
Vậy bài toán có thể biểu diễn dưới dạng:
$$a_1^2(b_2^2+c_2^2-a_2^2)+b_1^2(c_2^2+a_2^2-b_2^2)+c_1^2(a_2^2+b_2^2-c_2^2) \ge \sqrt{\left[\left(\sum_{cyc}a_{1}^2 \right)^2-2\sum_{cyc}a_{1}^4\right]\left[\left(\sum_{cyc}a_{2}^2 \right)^2-2\sum_{cyc}a_{2}^4 \right]}$$
Áp dụng BĐT Aczel,ta có:
$$VP \le \left(\sum_{cyc}a_{1}^2 \right).\left(\sum_{cyc}a_{2}^2 \right)-2\sqrt{\left(\sum_{cyc}a_{1}^4 \right)\left(\sum_{cyc}a_{2}^4 \right)}$$
Ta chỉ cần chứng minh:
$$2\sum_{cyc}a_1^2a_2^2 \le 2\sqrt{\left(\sum_{cyc}a_{1}^4 \right)\left(\sum_{cyc}a_{2}^4 \right)}$$
Bất đẳng thức trên đúng theo Cauchy-Schwart.
Vậy ta có điều phải chứng minh. $\blacksquare$
Cách 2: Phương pháp hình học giải tích
Giả sử 2 tam giác $A_1B_1C_1$ và $A_2B_2C_2$ có các đỉnh trong mặt phẳng có hệ trục tọa độ trực chuẩn là $A_1(0;p_1);B_1(p_2;0);C_1(p_3;0)$ và $A_2(0;q_1);B_2(q_2;0);C_2(q_3;0)$.
Sử dụng bất đẳng thức $x^2+y^2\ge 2|xy|$ ta có
$$a_1^2(b_2^2+c_2^2-a_2^2)+b_1^2(c_2^2+a_2^2-b_2^2)+c_1^2(a_2^2+b_2^2-c_2^2)$$
$$=(p_3-p_2)^2(2q_1^2+2q_1q_2)(p_1^2+p_3^2)(2q_2^2-2q_2q_3)+(p_1^2+p_2^2)(2q_3^2-2q_3q_2)$$
$$=2(p_3-p_2)^2q_1^2+2(q_3-q_2)^2p_1^2+2(q_2p_3-q_3p_2)^2\ge 2\begin{pmatrix}
(p_3-p_2)^2q_1
\end{pmatrix}^2+2\begin{pmatrix}
(q_3-q_2)^2p_1
\end{pmatrix}^2$$
$\geq 4|(p_3-p_2)q_1||(q_3-q_2)p_1|=16S_1S_2$ 
Vậy ta có điều cần chứng minh.
Copyright © 2012 -